What is Magnetic: Definition and 1000 Discussions

Magnetism is a class of physical attributes that are mediated by magnetic fields. Electric currents and the magnetic moments of elementary particles give rise to a magnetic field, which acts on other currents and magnetic moments. Magnetism is one aspect of the combined phenomenon of electromagnetism. The most familiar effects occur in ferromagnetic materials, which are strongly attracted by magnetic fields and can be magnetized to become permanent magnets, producing magnetic fields themselves. Demagnetizing a magnet is also possible. Only a few substances are ferromagnetic; the most common ones are iron, cobalt and nickel and their alloys. The rare-earth metals neodymium and samarium are less common examples. The prefix ferro- refers to iron, because permanent magnetism was first observed in lodestone, a form of natural iron ore called magnetite, Fe3O4.
All substances exhibit some type of magnetism. Magnetic materials are classified according to their bulk susceptibility. Ferromagnetism is responsible for most of the effects of magnetism encountered in everyday life, but there are actually several types of magnetism. Paramagnetic substances, such as aluminum and oxygen, are weakly attracted to an applied magnetic field; diamagnetic substances, such as copper and carbon, are weakly repelled; while antiferromagnetic materials, such as chromium and spin glasses, have a more complex relationship with a magnetic field. The force of a magnet on paramagnetic, diamagnetic, and antiferromagnetic materials is usually too weak to be felt and can be detected only by laboratory instruments, so in everyday life, these substances are often described as non-magnetic.
The magnetic state (or magnetic phase) of a material depends on temperature, pressure, and the applied magnetic field. A material may exhibit more than one form of magnetism as these variables change.
The strength of a magnetic field almost always decreases with distance, though the exact mathematical relationship between strength and distance varies. Different configurations of magnetic moments and electric currents can result in complicated magnetic fields.
Only magnetic dipoles have been observed, although some theories predict the existence of magnetic monopoles.

View More On Wikipedia.org
  1. Alfredo Tifi

    Synchrotron Radiation: Charge Loss of Relativistic Particles in Magnetic Fields

    Problem Statement: It is possible to describe synchrotron radiation as caused by a loss of electrical charge of relativistic particles that are moving in a magnetic field? Relevant Equations: E = mc2 An Italian expert of black hole M87 (Elisabetta Liuzzo) explains that the expected axial...
  2. Haorong Wu

    Why -e(v/c)H is the magnetic force?

    In a paper I am reading, it reads, "For these orbits the electric force ##-e E_r## almost balances the magnetic force ##-e \left ( v/c \right ) H_0##." where ##-e## is the charge of the electrons, ##v## is the speed of the electrons, ##H_0## is a homogeneous magnetic field, and ##c## is not...
  3. D

    50Hz 3 Phase Circuit - Twisting to reduce EMI?

    I have done the maths to work out the Induced Voltage on a communication cable, from a bunch of 3 phase circuits in a panel - with the minimum distance between them I calculated it to be about 1.5nV; but for a larger system where there could be larger loads on the cables I was thinking about...
  4. D

    I Earth’s Magnetic field formula or downloadable vector field

    I want to render the Earth’s Magnetic field in a software and simulate solar wind electron interaction with it. How do I calculate the magnetic strength and vector orientation at each point around the Earth up to thousands of km? Is there a formula? Or do I need to download a vector field from...
  5. Thurirl

    Creating slag through magnetic compression

    This is simply a top of the head idea that seems kinda cool but I don't really know what it could be good for. My idea was, would it be possible through the use of magnetic pressure can a metal be reduced to a slag or almost liquid like state? I know that more than likely it isn't possible due...
  6. N

    I need some help with Magnetic Fields & Computer Science

    Lets say we have a Direct Current (not an alternating current) going through a wire ; so there is a steady-state magnetic field created around the wire. Then let's say we move some object into the magnetic field that disturbs the field. . My question is : How do we/they get information...
  7. S

    Force on a copper loop entering into a magnetic Field B with speed v

    Hi, second problem in one evening, I'm sorry! But I'm also not quite sure if I did this one right. I had thought I need lenz's law but there is no current before entering the field so I just use the induced Voltage? My approach: ## V = \frac {B*A}{t} ## ## IR = \frac {B*A}{t} ## and ## A = v*t...
  8. Cyclone Charlie

    Particle in a magnetic field -- question

    I went with R=mv/qb, thus -> 6.64e-27*35.6e3/2*1.6e-19*1.8, and got 4.1e-4 m (metres), so diameter is 2R, 8.2e-4 m, as an answer, the reference site gives 3.95e+10 m as the answer, who's right here?
  9. C

    Magnetic properties of silver steel

    Hi, A straight question here, no complicated quantum theory equations for this, I just want to know a piece of data. What is the saturation flux of silver steel? It has a fraction of a percent of chromium in it, so was wondering if it was less than carbon steel's ~1.6T?
  10. A

    Guiding Magnetic Fields in Solenoid Coils: Design and Material Considerations

    Hi, at this moment I'm trying to figure out one thing. I have a solenoid with a core that has an empty middle, the flux normally loops back around the outside of the solenoid to the other side where it enters back into the core. I need to route this field between the two ends of the solenoid...
  11. D

    I E- & Ions Deflection by Earth's Magnetic Field

    Do solar wind electrons turn left and positively charged ions turn right if they are interacting weakly due to long distance with Earth's magnetic field and fail to complete a loop for the electrons that are on the left of Earth and ions on the right? I assume electrons on the right and ions on...
  12. S

    How to measure electric and magnetic fields in asymmetric capacitors?

    Summary: I need to build an asymmetric capacitor, but the mathematics of electromagnetics become too tough, do you have any info that can help? Appreciate it! Hello, I am an undergraduate student in engineering and I want to build an asymmetric capacitor, so I need electromagnetics which I...
  13. A

    Rigid high magnetic permeability materials

    Hi, for lower frequencies most electrical devices use laminated electric steel , like transformers and motors. But this makes the core made up of individual metal sheets and in older transformers taking the core apart sometimes results in the individual sheets falling apart. Is there any...
  14. Moara

    Eletromagnetism: Copper Plate on a Spring Oscillating in a Magnetic Field

    Tried to find the resultant force, but I can't see how the magnetic field affects. I used Faraday's law to find the the diferece of potentials in the plate Wich should be B.d.v, where v is the vertical velocity of plate, but there were not given the resistance or resistivity to relate with the...
  15. J

    Moving Electrons in a Uniform Magnetic Field

    a) We can solve for acceleration by looking at FNETy FNETy = FE (G is negligible) FNETy = m * a The mass (m) of an electron is 9.1093836 x 10-31 kg. The elementary charge (q) of an electron is -1.60217662 x 10-19 C a = ε * q / m a = (4.0 x 102 N/C * 1.6022 x 10-19 C) / 9.1094 x...
  16. G

    Where is the Error in My Derivation of the Magnetic Field Equation?

    As I said my goal is to derive the equation ##\tilde{B}^k(\vec{q})=-i\varepsilon^{ijk}q^i\tilde{A}^j_{cl}(\vec{q})## As far as I know, the magnetic field is defined using the potential as ##\vec{B}=\vec{\nabla}\times\vec{A}## Then in equation 6.6 they define ##A^\mu(x)=\int...
  17. Muhammad Usman

    How can a magnetic field generate a Potential Difference?

    Problem Statement: Potential difference is the difference in the quantity of charge in both potentials. How does the magnetic field creates this difference of charges ? Relevant Equations: None Hi, I was reading electricity and found that the difference in potential of both end is the...
  18. merlyn

    The phase of electric and magnetic forces in a photon.

    I'm curious if someone help me understand why the electric and magnetic forces are IN PHASE at right angles to one another? Should they not be 90 degrees out of phase in order to conserve energy? I do understand they are in phase but why? Thank you all for your time. Merlyn.
  19. M

    Magnitude of the Magnetic Field near a Circuit Inclined at an Angle

    For the front wire, I got the magnitude of the magnetic field in terms of the magnitude of the magnetic force, the current, "l," and the "theta". I am unsure how to proceed because I thought that the magnetic force is independent of any other forces. I am also just lost in general. Any help...
  20. H

    I A bar magnet and magnetic moment in a magnetic field

    Why a magnet bar would become aligned with the external magnetic field while a magnetic moment would precess around the external magnetic field?
  21. Z

    Eigenstates of a free electron in a uniform magnetic field

    I started with the first of the relevant equations, replacing the p with the operator -iħ∇ and expanding the squared term to yield: H = (-ħ^2 / 2m)∇^2 + (iqħ/m)A·∇ + (q^2 / 2m)A^2 + qV But since A = (1/2)B x r (iqħ/m)A·∇ = (iqħ / 2m)(r x ∇)·B = -(q / 2m)L·B = -(qB_0 / 2m)L_z and A^2 =...
  22. Physics lover

    Motion of a particle in a uniform magnetic field

    All i could think is that the z component of velocity will remain unchnged as there is no force in that direction.And for the x and y component can we imagine the helical motion as a superposition of a circle and a straight line.So for x and y component can we solve for a particle moving in a...
  23. DaniV

    Finding the magnetic field inside a material shell under external field

    I know that we need to use some boundry condition both on the a radius surface and the b radius surface and somehowuse the superposition on them both, the boundry condition most be for the tangential and the radial part, the only things I got is that i don`t know how to produce a magnetisation M...
  24. JD_PM

    What is the significance of neglecting the encircled sides in Ampere's law?

    This is a very basic issue but really important as well. The rectangular loop has length ##l## and width ##h##. I have seen the argument of neglecting the encircled sides of the loop because ##h << 1## while using Ampere's law to calculate the magnetic field flowing over a plane. I find this...
  25. D

    I Electromagnetic Fields and the manipulation of Space-Time

    Summary: Mathematical and Physical queries in regards to Electromagnetic Fields and their manipulation of Space-Time. I recently started looking into Einstein's Field Equations, to get a better understanding of how mass distorts and curves the plane of Space-Time, however from doing this I...
  26. P

    How to calculate the voltage induced in a coil by an AC magnetic field?

    So, as it says in the title, I am trying to calculate overall voltage induced in a coreless coil in the cases of it being stationary and moving in an alternating magnetic field. To go more into detail, I would like to create a mathematical model of a coil in an alternating magnetic field that...
  27. DaniV

    Magnetic Field around two magnetic boxes

    I tried to look once at the zy axis and saw a two infinite capacitors with fictive charge density of M on the upper plane, and -M in the lower with a distance of h from each other, the two capacitors saparated with d in the y axis, but when I look in xy axis there was 2 another capcitors the...
  28. A

    Conductive loop that is contracting in a magnetic field

    $$V = \int \left(\vec{v} \times \vec{B}\right) \bullet \vec{dl} - \int _S \frac{\vec{dB}}{dt} \bullet \vec{ds}$$ From the statement I know that: B⊥v, (B x v) // dl and B // ds. $$V = \int vBdl - \oint _S \frac{dB}{dt} ds$$ v is the speed with which all the segments dl are aproximating to the...
  29. L

    Usage of absorption and magnitude mode spectra in nuclear magnetic resonance

    In nuclear magnetic resonance, when should one process spectra in absorption mode and when - in magnitude mode? What is benefit of using the first one and the second one? For example, I need to integrate spectrum. In this case, what should I use - absorption or magnitude mode?
  30. fight_club_alum

    Exploring Magnetic Forces on Current: A Visual Guide

    This is how I visualize the problem (of course I am drawing this as if it is in the z-y axis); I don't know what will be the next step. Anyone please help me. Thank you
  31. K

    Cell Metabolism and Static, Unidirectional Magnetic Fields

    Can anyone explain why an artificial magnetic field may or may not promote an increase in cellular metabolism?
  32. B

    Do magnetic lenses do work on particles flowing through them?

    Do magnetic lenses do work on charged particles flowing through them? Intuitively I would think yes because the magnetic field produced by the electric current in the coil is applying a force on the particles flowing through the lens, so therefore an electric current (the beam of particles to...
  33. K

    Magnetic field around a conductor with protons?

    Assuming that you can create a proton current. For example, the current of ionized hydrogen is analogous to a conductor. Question! Will a magnetic field be created around a conductor with a current of protons? By analogy with the magnetic field of electrons in a conductor.
  34. fight_club_alum

    Particle in a circular path due to magnetic field

    v = sqrt( (2 * charge of proton * 3000/e) / (mass of proton)) v = 1.893986024 x 10^`15 r = ( (mass of proton) * (velocity) ) / ((magnetic field) * (charge of proton)) r = 24715769.68 m Anyone please help
  35. T

    Expressing the magnetic vector potential in terms of its curl

    We have the retarded magnetic vector potential ##\mathbf{A}(\mathbf{r},t) = \dfrac{\mu_0}{4\pi} \int \dfrac{\mathbf{J}(\mathbf{r}',t_r)}{|\mathbf{r}-\mathbf{r}'|} \mathrm{d}^3 \mathbf{r}'## And its curl ##\mathbf{B}(\mathbf{r}, t) = \frac{\mu_0}{4 \pi} \int \left[\frac{\mathbf{J}(\mathbf{r}'...
  36. fight_club_alum

    Path diameter difference of two singly-charged ions in a magnetic field

    M1(50,000)/q (0.4) - M2(50,000)/q (0.4) = 0.025 Is there a special charge for singly charged ions?
  37. fight_club_alum

    Particle that moves in an electric and magnetic field

    m = 0.005 q = -70 x 10^-6 c v = 30,000 m/s Since there is no movement vertically Fb = Mg So, q . V . B = mg So, (70 x `10^-6) . (30,000) . B = (0.005) . (9.8) So, B = 0.0233333 or ~ 23 MT
  38. N

    Magnetic attraction and repulsion equivalence

    Equations of attraction or repulsion can get very complicated when the field shapes and densities are not identical. Intuition would hold the forces to be the same, but in on closer examination the field shape of two repulsing magnets looks entirely different from two attracting magnets. I...
  39. K

    Find ∆V while sliding a card through a card reader with magnetic field

    emf = dΦ/dt = (B*A)*d/dt = B(dA/dt), dA/dt= L*d/dt(vt) = L*v, emf = B*L*v per coil Since there are 25 loops the total emf= 25(vBL) This is where I'm am stuck. Would I assume that B is 24 uT, the velocity as 3m/s , and the length as 1mm? If so I would get ∆V as 1.8*10^-6.
  40. K

    Strength of magnetic field at the end of a wire wrapped nail.

    V=I*R 6v=I*(0.6+0.9)ohms I=4amp B=100*(uo)(2N)(I)/L * 1/2 I think since the wire is double wrapped, we multiply the equation by 2, but since we are looking for the magnetic field at the end of the wire we also have to multiply the equation by 1/2 I=4A, uo= 4pi*10^-7 2N/L turns per unit...
  41. fight_club_alum

    Calculating Magnetic Force with Vectors

    8000 0 0 5 -4 3 F = (5 x 10^-6) * up (sorry can't write a materix here) F = (5 x10^-6) {0i - 24000 j - 32000 k) F = -0.12 j - 0.16 k Mag of F = 0.2 (I feel that there is something wrong in the question; I don't know)
  42. fight_club_alum

    Find the charge of this particle moving in a magnetic field

    F = ma F = (6x10^-6) * 8 F = 4.8 * 10^-5 F = QBVsin(theta) F/(BVsin(theta) = Q (4.8 x 10^-5) / (5 x 10^-3) (4000) (sin(37)) = 3.98 x 10^-6 ~ 4 uc <---- THE RIGHT ANSWER IS -4 uc
  43. dRic2

    Magnetic Monopole: Reason Behind Faraday's Law Counterpart

    For the magnetic fields it is obvious that ##F = q_m B##, but I don't get why the final result is $$\mathbf F = q_m(\mathbf B -\frac 1 {c^2} \mathbf v \times \mathbf E)$$ The second part is like a "counterpart" of Faraday's Law, but I do not understand why it should be there... For what reason...
  44. H

    Work performed by a magnetic field

    Here is a little thought experiment related to magnetism and a perplexing question regarding its physics. Suppose we have a long cylinder of transparent plastic, and we press fit and then cement a circular magnet in one end of the cylinder with its north pole oriented into the cylinder. We also...
  45. Wayne Lai

    Acceleration of a metal piece due to dipole radiation magnetic field

    Recently I am learning about electrodynamic radiation and its various types, and it occur to me that since the form of the magnetic field created by the dipole radiation is some combination of cos(wt), 1/r, and cos(kr) (take the approximation of r >> c/w) Therefore, if there is a metal placed...
  46. N

    Magnetic force on a wire due to a loop

    I just need to confirm my intuition that the magnetic force on the wire carrying I2 is 0. Basis for my intuition: * Right above the center of the loop carrying I1, the magnetic field lines are in exactly the same direction as the piece of wire carrying I2, so [a] x [B1] = 0. (photo from...
  47. Y

    Magnetic force -- does it fall off to zero far away from the source?

    Let's say we have a wire and a circuit is closed the magnetic field start to spread with speed of light c. Then we would have a place " out of range " at which the magnetic field doesn't exist.What is the equation that connects magnetic force with distance with its prorogation of speed of light...
  48. themountain

    The magnitude and direction of the minimum magnetic field

    F=ILB F=(48)(.76)B F/36.48=B I am stuck at how to find F, is there a different formula I am missing?
Back
Top